Akademisyenler öncülüğünde matematik/fizik/bilgisayar bilimleri soru cevap platformu
0 beğenilme 0 beğenilmeme
985 kez görüntülendi

"$(E_n)$ dizisi yakınsaktır. $\Leftrightarrow$ sonsuz çokluktaki n ler için $x\in E_n$ ve $x\in X\backslash E_n$ olacak şekilde $x\in X$ noktaları yoktur." önermesinin doğruluğunu gösteriniz.

Lisans Matematik kategorisinde (470 puan) tarafından  | 985 kez görüntülendi

$x\in E_n$ ve $x\in X$\ $E_n$?

aynen bu şekilde verilmiş soruda

$(E_n)$'in yakinsamasi ne demek tam olarak?

Burada $X$ bir küme, $(E_n)$ de $X$ in alt kümelerinin bir dizisi oluyor.

$\lim \sup E_n= \bigcap\limits_{m=1}^{\infty}(\bigcup\limits_{n=m}^{\infty}E_n)$

$\lim \inf E_n= \bigcup\limits_{m=1}^{\infty}(\bigcap\limits_{n=m}^{\infty}E_n)$ 

olmak üzere $\lim \sup E_n=\lim \inf E_n=E$ ise $(E_n)$ dizisi $E$ kümesine yakınsar deniyor. Tanımı bu benim bildiğim.

Tesekkurler. Sanirim soruda $X \setminus E_n$ yerine $X \setminus E_{n+1}$ dersek sorunu cozuyoruz. Tanimlarla biraz oynayarak, 

  • $x \in \limsup E_n$ ancak ve ancak sonsuz sayida $n$ icin $x \in E_n$ ise, ve
  • $x \in \liminf E_n$ ancak ve ancak bir sure sonra her $n$ icin $x \in E_n$ ise
oldugunu gorebiliriz.
$\liminf E_n \subseteq \limsup E_n$ oldugu kolayca gozukuyor boyle bakildiginda.
Esitligin olmadigini, yani dizinin yakinsamadigini varsayalim. $x \in X$ elemani sonsuz sayida $n$ icin $E_n$ kumelerinde yer alsin ama bir sure sonra her $E_n$'de yer aliyor olmasin. Yani surekli soyle bir durumla karsilasiyoruz: $x \in E_n$ olacak sekilde bir $E_n$ buluyoruz surekli ama bir sure sonra oyle bir $m$ geliyor ki $x \notin E_m$ oluyor. Bu $n$'ler ve $m$'lerle oynayarak $m = n +1$ oldugunu kabul edebiliriz. Toparlayacak olursak, $E_n$ yakinsak degilse sonsuz sayida $n$ icin $x \in E_n$ ve $x \notin E_{n+1}$ (baska bir deyisle $x \in X \setminus E_{n+1}$) olacak sekilde bir $x \in X$ varmis.
Ayni sekilde, boyle bir $x$ olmamasi demek de su demek: Bir $x \in X$, sonlu sayida $n$'den sonra bir $E_n$'de yer aliyorsa, $E_{n+1}$'de de yer almalidir. O halde, bir sure sonra butun $E_n$'lerde yer almalidir (tumevarim). Yani, $\limsup$, $\liminf$'e esittir.


Çok teşekkür ederim.

$p$ : Limit vardir.

$q$ : Boyle bir $x$ yoktur. 

Ispat etmemiz gereken: $p \iff q$

Ben ne ispatladim:

Ilk once, $\neg p \implies \neg q$ oldugunu soyledim. Yani, limit yoksa boyle bir $x$ vardir dedim.

Sonra, $q \implies p$ oldugunu soyledim. Boyle bir $x$ yoksa, limit vardir dedim.

Ama bu ikisi ayni sey! Yani ayni seyi iki kere kanitlamis oldum.

Yani, yukarida vermis oldugum kanit sadece bir yonu kanitliyor. Diger yon hala kanitlanmadi. Ama bu tarz arumanlarla kanitlamasi zor degil.


20,200 soru
21,727 cevap
73,275 yorum
1,887,846 kullanıcı